Integration problem using Integration by Parts

Click For Summary
The discussion revolves around an integration problem that involves using integration by parts and exploring alternative methods. A function f(x) is defined, and it is established that its derivative f'(x) equals zero, indicating that f(x) is constant. The participants suggest calculating the constant C from the definition of f(x) and note that only C=0 may be possible. Additionally, a Weierstraß substitution is mentioned as a potential alternative, though its effectiveness is questioned. The conversation highlights the importance of careful differentiation and integration limits in solving the problem.
chwala
Gold Member
Messages
2,827
Reaction score
415
Homework Statement
The integral ##I_{n}##, where ##n## is positive is given by ##I_{n}=\int_{0.5}^1x^{-n} sinπx\, dx##

show that,

##n(n+1)I_{n+2}=2^{n+1} + π-π^2 I_{n}##
Relevant Equations
integration by parts...
1594298971707.png


i would appreciate alternative method...
 
Physics news on Phys.org
You can differentiate to check it, but watch out for the integration limits, as differentiation isn't exactly the opposite of integration.
 
hi fresh...long time...ok you may direct me...i am trying to refresh in this things...
 
Let's define ##f(x)=n(n+1)I_{n+2}-2^{n+1}+\pi - \pi^2 I_n##. Then show that ##f'(x)=0## which means ##x \longmapsto f(x)## is constant, say ##f(x)=C##. At last calculate ##C## from the definition of ##f(x)##, or show that only ##C=0## is possible.

I haven't done it. so I don't know how it will work. The entire question is a standard example for integration by parts. Another possibility could be using a Weierstraß substitution, but I don't think this would change a lot.
 
I_n is the imaginary part of <br /> J_n = \int_{1/2}^1 x^{-n} \mathrm{e}^{\mathrm{i} \pi x}\,dx<br /> and you can establish the result* by interating this by parts once to get a relation between J_{n+2} and J_{n+1}, and you can then substitute for J_{n+1} in terms of J_n before taking imaginary parts.

*So far as I can tell, there should be an n multiplying 2^{n+1}. chwala's work agrees with this until the last line, where it mysteriously disappears upon multiplying (n+1)I_{n+2} by n.
 
  • Like
Likes SammyS
Question: A clock's minute hand has length 4 and its hour hand has length 3. What is the distance between the tips at the moment when it is increasing most rapidly?(Putnam Exam Question) Answer: Making assumption that both the hands moves at constant angular velocities, the answer is ## \sqrt{7} .## But don't you think this assumption is somewhat doubtful and wrong?

Similar threads

  • · Replies 1 ·
Replies
1
Views
2K
  • · Replies 3 ·
Replies
3
Views
2K
  • · Replies 5 ·
Replies
5
Views
1K
  • · Replies 11 ·
Replies
11
Views
3K
Replies
2
Views
1K
  • · Replies 4 ·
Replies
4
Views
2K
  • · Replies 13 ·
Replies
13
Views
2K
  • · Replies 19 ·
Replies
19
Views
2K
  • · Replies 12 ·
Replies
12
Views
2K
  • · Replies 5 ·
Replies
5
Views
2K